Variación de término como ∂RabRab∂Rabcd∂RabRab∂Rabcd\frac{\parcial R_{ab} R^{ab}}{\parcial R_{abcd}}, ∂RabcdRabcd∂Refgh∂RabcdRabcd∂Refgh\frac{\parcial R_ {abcd} R^{abcd}}{\parcial R_{efgh}}

Esto está relacionado con mi pregunta anterior Variación con respecto a R a b C d ? como calcular R R a b C d = 1 2 ( gramo a C gramo b d gramo a d gramo b C ) ? En este caso me gustaría calcular el tensor de Ricci

R a b R a b R a b C d =

En este caso, ¿cómo calculo las derivadas?

Y más para

R a b C d R a b C d R mi F gramo h
¿Puedo usar derivados anteriores? ( X 2 ) = 2 X , y decir lo anterior como 2 R mi F gramo h ?

Respuestas (2)

Empezamos por señalar que

R a b R C d mi F = ( R C a mi b gramo C mi ) R C d mi F = ( R C d mi F d a d d b F gramo C mi ) R C d mi F ,
sugiriendo una respuesta en la línea de d a d d b F gramo C mi . Pero por las propiedades de antisimetría del tensor de Riemann, hay más de una forma de escribir R a b como una contradicción de R C d mi F con un tensor.

Necesitamos una antisimetría al intercambiar C con d , sugiriendo una respuesta en la línea de 1 2 ( d a d d b F gramo C mi d a C d b F gramo d mi ) . Pero eso tampoco puede ser del todo correcto: también necesitamos una antisimetría al intercambiar mi con F , sugiriendo una respuesta en la línea de 1 4 ( d a d d b F gramo C mi d a C d b F gramo d mi d a d d b mi gramo C F + d a C d b mi gramo d F ) . Pero todavía necesitamos C d mi F mi F C d ser una simetría, dando el resultado final

R a b R C d mi F = X a b C d mi F := 1 8 ( ( d a d d b F + d a F d b d ) gramo C mi ( d a C d b F + d a F d b C ) gramo d mi ( d a d d b mi + d a mi d b d ) gramo C F + ( d a C d b mi + d a mi d b C ) gramo d F ) .

Tenga en cuenta que cada término tiene a b como índices más bajos y C d mi F como índices superiores.

Por la regla del producto,

( R a b R a b ) R C d mi F = R a b R C d mi F R a b + R a b R a b R C d mi F .
Podemos cambiar las alturas de a , b en el segundo término, a saber.
( R a b R a b ) R C d mi F = 2 R a b X a b C d mi F .
Expresiones como R a b d a d d b F gramo C mi = gramo C mi R d F dar
( R a b R a b ) R C d mi F = 1 2 ( gramo C mi R d F gramo d mi R C F gramo C F R d mi + gramo d F R C mi ) .
Tenga en cuenta que cada término tiene C d mi F como índices superiores y a b no existen en el lado derecho, ya que son índices ficticios contraídos en el lado izquierdo.

Para la segunda derivada, imagina que en su lugar quisiéramos ( V a V a ) V b para un vector; la respuesta seria 2 V b , sugiriendo una respuesta como 2 R mi F gramo h . Esto ya tiene las propiedades correctas, así que hemos terminado.

¡Gracias!. Hice otra publicación, physics.stackexchange.com/questions/301884/… . En este momento estoy tratando de variar los productos de dos tensores de Riemann contraídos. Si te interesa por favor haz algún comentario.

Del útil comentario @JG,

( R a b R a b ) R C d mi F = 2 R a b X a b C d mi F , X a b C d mi F = R a b R C d mi F

Trate de calcular

R m v = R a b C d gramo b d d m a d v C = 1 8 R a b C d ( gramo b d d m a d v C + gramo b d d m C d v a gramo a d d m b d v C gramo a d d m C d v b gramo b C d m d d v a gramo b C d m a d v C + gramo a C d m d d v b + gramo a C d m b d v d )
donde i antisimetrizó (a,b) y (c,d) y simetrizó los pares (ab, cd), y simetrizó m , v en paréntesis lateral

Así que supongo que

X m v a b C d = R m v R a b C d = 1 8 ( gramo b d d m a d v C + gramo b d d m C d v a gramo a d d m b d v C gramo a d d m C d v b gramo b C d m d d v a gramo b C d m a d v C + gramo a C d m d d v b + gramo a C d m b d v d )

R m v R m v R a b C d = R a [ C gramo b ] d + R b [ d gramo a ] C = 1 2 ( R a C gramo b d R b C gramo a d R a d gramo C b + R b d gramo C a )

¿Tengo razón?

Escribiré una respuesta pronto. Tu fórmula para X no tiene sentido; tiene dos índices de bonificación, además coloca algunos de los índices a la altura incorrecta.
@JG, lo siento, hay algunos errores tipográficos. lo corrijo
@JG, después de corregir los índices correctamente, obtengo la misma respuesta en su publicación. Gracias.